LSAT and Law School Admissions Forum

Get expert LSAT preparation and law school admissions advice from PowerScore Test Preparation.

 studyhelp20
  • Posts: 28
  • Joined: Dec 09, 2020
|
#82128
Can you please explain the correct answer for this question and explanations of the incorrect answers? thanks
 Adam Tyson
PowerScore Staff
  • PowerScore Staff
  • Posts: 5153
  • Joined: Apr 14, 2011
|
#82359
For a Must Be False question, look for something impossible, studyhelp20. The key to this one is the JG block - because of that rule, either J is Wednesday and G is Thursday, or else J is Thursday and G is Friday. No matter what, either J or G MUST be on Thursday, so there is no way that H is the only meeting on Thursday, making answer C impossible and therefore the correct answer.

A. H can be the only meeting on Wednesday. That would put the JG block on Thursday and Friday, and then to ensure that K and G are close enough to each other to satisfy the last rule of the game we would put them both on Friday (doesn't matter which is morning and which is afternoon). L is forced into Thursday morning, putting J in the afternoon.
B. G can be all alone on Thursday - see the templates in the main setup thread.
D. K can be the only one on Friday (and in fact that happened in question 20, so use your prior work to quickly eliminate this answer)
E. L can be Friday morning - again, the templates will prove that

I wouldn't go through testing every answer. I would scan the answers, sorting losers and contenders and looking for obvious trouble, and answer C is an obvious problem due to the JG block. Seeing that, I would suggest you pick that answer and move on without further delay!

Get the most out of your LSAT Prep Plus subscription.

Analyze and track your performance with our Testing and Analytics Package.